List the next 4 multiples for the number 21.

Answers

Answer 1

Answer:

21,42,63,84,105 126,147,168,189, and 210.

I hope this helps.


Related Questions

3. Mr. Black's pay increased by 15% when he got a raise, bringing his new salary to $184. What was Mr. Black's original pay? ​

Answers

Answer:

$160

Step-by-step explanation:

115% = $184

100% = X

let X be the original salary

(X is unknown)

115% = $184

100% = X

cross mutiply

115 x X = 100 x 184

115X = 18400

X = 18400 divided by 115

X = $160

therefore,Mr Black's original salary before increased by 15% was $160

hope am helpful.

184/x = 115/100
184*100=18400
18400=115x
18400/115=160
x=160

Find the difference between Simple interest and compound interest on 1200 for one year at 10% per annum​

Answers

Answer:

the difference between them is zero cause on first year simple interest and compound interest are equal

Nathan shares out 12 sweets he gives yasmin 1 sweet for 3 sweets he buys how many sweets does nathan get

Answers

Given that, Nathan shares out 12 sweets, if he gives Yasmin 1 sweet for 3 sweets he buys, Nathan will get 9 sweets.

How many sweets does Nathan gets?

Given that, Nathan shares out 12 sweets, he gives Yasmin 1 sweet for 3 sweets he buys.

Let the sweet be represented by x

For each x sweet for Yasmin, Nathan gets 3x sweets

Hence

x + 3x = 12

We solve for x

4x = 12

x = 12/4

x = 3

Hence;

Yasmin gets x sweet = 3

Nathan gets 3x sweets = 3 × 3 = 9

Given that, Nathan shares out 12 sweets, if he gives Yasmin 1 sweet for 3 sweets he buys, Nathan will get 9 sweets.

Learn more word problems here: brainly.com/question/14539651

#SPJ1


lim 1 − cos(2x) /x x→0

Answers

The value of the limit of the function given when the value of x tends to zero is 0

What is a limit of a function?

The limit of a function is a fundamental concept in calculus and analysis concerning the behavior of that function near a particular inputs.

Given the function limit below;

lim x→0 [1 − cos(2x) /x]

Substitute the value of x into the expression to have;

f(0) = 1-cos2(0)/0
f(0) = 1-1/0

f(0) = 0/0 (ind)

Apply the L'hospital rule to have:

lim x→0 [ − (-2sin2x)) /1]

Substitute the value of x into the result

f(0) = 2sin2(0)/1

f(0) = 2(0)

f(0) = 0

Hence the value of the limit of the function given when the value of x tends to zero is 0

Learn more on limit of a function here: https://brainly.com/question/23935467

#SPJ1

Determine the number of solutions for the equation shown below.
4x+6= 4x+6
Ο Α. 0
OB. Infinitely many
* O C. 1
OD. 2

Answers

B) it’s infinite you can put -30 in as x and still get the same answer on the other side you could also put x as zero and get 6 as an answer the answer is infinite

a. How many bushels of corn, if any, will the United States export or import at the prices below?
At $1, it will (import, export, or neither import or export )
___bushels.

b. How many bushels of corn, if any, will the United States export or import at the prices below?
At $2, it will (import, export, or neither import or export )
____bushels.

c. How many bushels of corn, if any, will the United States export or import at the prices below?
At $3, it will (import, export, or neither import or export )
____ bushels.

d. How many bushels of corn, if any, will the United States export or import at the prices below?
At $4, it will (import, export, or neither import or export )
____ bushels.

e. How many bushels of corn, if any, will the United States export or import at the prices below?
At $5, it will (import, export, or neither import or export )
____ bushels.

2.

a. Use this information to construct the U.S export supply curve and import demand curve for corn

b. Suppose that the only other corn-producing nations is France, where the domestic price is $4. Which country will export corn and which country will import it

The United Stated will import corn France will export it

The United States will export corn and France will import it

Neither country will import or export corn.

Answers

At $1, the United States will import 15,000 bushels of corn.At $2, the United States will import 7,000 bushels of corn.At $3, the United States will neither import nor export any bushels of corn.At $4, the United States will export 6,000 bushels of corn.At $5, the United States will export 10,000 bushels of corn.

The number of bushels of corn.

Based on the graph which depicts the United States domestic market for  corn at different prices, we can logically deduce the following information:

At $1, the United States will import 15,000 bushels of corn.At $2, the United States will import 7,000 bushels of corn.At $3, the United States will neither import nor export any bushels of corn.At $4, the United States will export 6,000 bushels of corn.At $5, the United States will export 10,000 bushels of corn.

Assuming France is the only other corn-producing nations at a domestic price of $4, we can reasonably infer that the United States will export corn while France will import it.

Read more on import here: https://brainly.com/question/24190346

#SPJ1

The diameter of an electric cable is normally distributed, with a mean of 0.9 inch and a standard deviation of 0.02 inch. What is the probability that the diameter will exceed 0.92 inch? (You may need to use the standard normal distribution table. Round your answer to three decimal places.)

Answers

The probability that the diameter of the electric cable that is normally distributed will exceed 0.92 inch is 0.841

What is probability?

Probabilities are used to determine the chances, likelihood, possibilities of an event or collection of events

How to determine the probability?

The given parameters are:

Mean = 0.9

Standard deviation = 0.2

Calculate the z-score at x = 0.92 using

[tex]z = \frac{x - \mu}{\sigma}[/tex]

This gives

z = (0.92 - 0.9)/0.02

Evaluate the numerator; subtract 0.9 from 0.92

z = 0.02/0.02

Divide 0.02 by 0.02. This gives

z = 1

The probability is then represented as:

P(x > 0.92) = P(z > 1)

Next, we look up the value of the z table of probabilities

From the z table of probabilities, we have:

P(x > 0.92) = 0.841

Hence, the probability that the diameter of the electric cable that is normally distributed will exceed 0.92 inch is 0.841

Read more about probability at:

https://brainly.com/question/11234923

#SPJ1

Please help quick!!!

Reflect over the y axis then translate

Answers

The composition of transformation that maps ABCD to EHGF is reflect over the y-axis, then translate (x - 1, y + 1)

What is transformation?

Transformation is the movement of a point from its initial location to a new location. Types of transformations are reflection, rotation, translation and dilation.

Translation is the movement of a point either up, left, right or down in the coordinate plane.

The composition of transformation that maps ABCD to EHGF is reflect over the y-axis, then translate (x - 1, y + 1)

Find out more on transformation at: https://brainly.com/question/4289712

#SPJ1

What is the value of tan M?
15
15
17
L
∞ ∞01 200
8
17
8
15
15
K 8
8
M

Answers

Answer:

The value of tan M = 15/8

Step-by-step explanation:

[tex]tan \: m \: = \frac{p}{b} \\ tan \: m \: = \frac{15}{8} [/tex]

Question 2 of 5
Select the correct answer.
After buying a car, Sarah decides to get it appraised every few years. After owning the car for two years, its value is $5,000. After own
the car for five years, its value is $2,000.
What is the equation that models this inverse variation?
Oy=
O y = 2,500
O
O
5,000
y = 2,000
y =
10,000
Submit
Reset

Answers

The inverse proportional relationship that models this variation is given as follows:

[tex]y = \frac{10,000}{x}[/tex]

What is a proportional relationship?

A proportional relationship is a function in which the output variable is given by the input variable multiplied by a constant of proportionality, that is:

y = kx

In which k is the constant of proportionality.

An inverse proportional relationship is given as follows:

[tex]y = \frac{k}{x}[/tex]

In this problem, we have an inverse relation in which y(2) = 5000, hence the constant k is found as follows:

[tex]y = \frac{k}{x}[/tex]

[tex]5000 = \frac{k}{2}[/tex]

k = 10,000

Hence the relation is:

[tex]y = \frac{10,000}{x}[/tex]

As stated in the problem, when x = 5, y = 2,000, which we can verify replacing in the relation.

More can be learned about proportional relationships at https://brainly.com/question/10424180

#SPJ1

How do you solve for m?

m= −(4+m)+2

Answers

Answer:

[tex]\bf m = - 1[/tex]

Step-by-step explanation:

[tex]\bf m= −(4+m)+2[/tex]

First of all, let's Rearrange the given terms :-

[tex]\bf m = - ( m +4) + 2[/tex]

Now, let's distribute:-

[tex]\bf m = - m - 4 + 2[/tex]

Add numbers -4 + 2 = -2

[tex]\bf m = - m - 2[/tex]

Add m to both sides, and combine like terms :-

[tex]\bf 2m = - 2[/tex]

Divide both sides by 2 :-

[tex]\bf m = - 1[/tex]

m= -(4+m)+2
m= -4-m+2
2m= -2
m= -1

Hope it helps : )

The number of marriage licenses issued by Clark County Nevada, the county where Las Vegas is located, has been
decreasing since the year 2000:
According to the model, how many marriage licenses were issued in 2003? Round your answer to the nearest hundred.
You must find the linear regression equation first.

Answers

The number of marriage licenses were issued in 2003 is 21586.

According to the statement

we have to find the number of marriage licenses were issued in 2003

and the given equation is y=3.405x^2-17674x+21533000

And in this equation x represent the time

And Y represent the number of marriage certificate issued.

So, in given equation

y=3.405x^2-17674x+21533000

we fill the value of X and find the value of Y means tne number of marriage license is issued in 2003

We know that X = 3 from 2000 to 2003

Then put X=3 in the given equation.

Then

This is the equation (1) represent the change

y=3.405(3)^2-17674(3)+21533000 -(1)

y = 30.645+53022+21533000

y = 21586052.645

On nearest hundred the value becomes 21586.

So, The number of marriage licenses were issued in 2003 is 21586.

Learn more about NUMBERS here https://brainly.com/question/1770447

DISCLAIMER: The question was incomplete. please find the full content below.

QUESTION:

According to the model, how many marriage licenses were issued in 2003? Round your answer to the nearest hundred.

The number of marriage licenses issued by Clark county Nevada, the county where Las Vegas is located, has been decreasing since the year 2000. This can be modeled by y=3.405x^2-17674x+21533000 where x is the year and y is the number of marriage licenses issued.

#SPJ1

Answer: A. 137,000

Step-by-step explanation: I did the quiz.

1/2x + 1/4y = 3 solve for y

Answers

Answer:

y=-2x+12

Step-by-step explanation:

First, subtract 1/2 from each side

1/4y = -1/2x + 3 --> multiply each side by 4

y = -2x + 12 is your final answer

how to find n if sn=969 of A.p,a1=9 and d=6​

Answers

‍‍‍‍‍‍‍‍‍‍‍‍‍‍‍‍‍‍‍‍‍‍‍‍‍‍‍‍‍‍‍‍‍‍ ‍ ‍‍‍‍‍‍‍‍‍‍‍‍‍‍‍‍‍‍‍‍‍‍‍‍‍ ‍ ‍ ‍ ‍ ‍‍‍‍‍‍‍‍‍‍‍‍‍‍‍‍‍‍‍‍‍‍‍‍ ‍ ‍ ‍ ‍ ‍‍‍‍‍‍‍‍‍‍‍‍‍‍‍‍‍‍‍‍‍‍‍‍ ‍ ‍ ‍ ‍ ‍‍‍‍‍‍‍‍‍‍‍‍‍‍‍‍‍‍‍‍‍‍‍‍ ‍ ‍ ‍ ‍ ‍‍‍‍‍‍‍‍‍‍‍‍‍‍‍‍‍‍‍‍‍‍‍‍ ‍ ‍ ‍ ‍‍‍‍‍‍‍‍‍‍‍‍‍‍‍‍‍‍‍‍‍‍‍‍‍ ‍ ‍‍‍‍‍‍‍‍‍‍‍‍‍‍‍‍‍‍‍‍‍‍‍‍‍ ‍ ‍ ‍ ‍ ‍‍‍‍‍‍‍‍‍‍‍‍‍‍‍‍‍‍‍‍‍‍‍‍ ‍ ‍ ‍ ‍ ‍‍‍‍‍‍‍‍‍‍‍‍‍‍‍‍‍‍‍‍‍‍‍‍ ‍ ‍ ‍ ‍ ‍‍‍‍‍‍‍‍‍‍‍‍‍‍‍‍‍‍‍‍‍‍‍‍ ‍ ‍ ‍ ‍ ‍‍‍‍‍‍‍‍‍‍‍‍‍‍‍‍‍‍‍‍‍‍‍‍ ‍ ‍ ‍ ‍‍‍‍‍‍‍‍‍‍‍‍‍‍‍‍‍‍‍‍‍‍‍‍‍ ‍ ‍‍‍‍‍‍‍‍‍‍‍‍‍‍‍‍‍‍‍‍‍‍‍‍‍ ‍ ‍ ‍ ‍ ‍‍‍‍‍‍‍‍‍‍‍‍‍‍‍‍‍‍‍‍‍‍‍‍ ‍ ‍ ‍ ‍ ‍‍‍‍‍‍‍‍‍‍‍‍‍‍‍‍‍‍‍‍‍‍‍‍ ‍ ‍ ‍ ‍ ‍‍‍‍‍‍‍‍‍‍‍‍‍‍‍‍‍‍‍‍‍‍‍‍ ‍ ‍ ‍ ‍ ‍‍‍‍‍‍‍‍‍‍‍‍‍‍‍‍‍‍‍‍‍‍‍‍ ‍ ‍ ‍ ‍‍‍‍‍‍‍‍‍‍‍‍‍‍‍‍‍‍‍‍‍‍‍‍‍ ‍ ‍‍‍‍‍‍‍‍‍‍‍‍‍‍‍‍‍‍‍‍‍‍‍‍‍ ‍ ‍ ‍ ‍ ‍‍‍‍‍‍‍‍‍‍‍‍‍‍‍‍‍‍‍‍‍‍‍‍ ‍ ‍ ‍ ‍ ‍‍‍‍‍‍‍‍‍‍‍‍‍‍‍‍‍‍‍‍‍‍‍‍ ‍ ‍ ‍ ‍ ‍‍‍‍‍‍‍‍‍‍‍‍‍‍‍‍‍‍‍‍‍‍‍‍ ‍ ‍ ‍ ‍ ‍‍‍‍‍‍‍‍‍‍‍‍‍‍‍‍‍‍‍‍‍‍‍‍ ‍ ‍ ‍ ‍‍‍‍‍‍‍‍‍‍‍‍‍‍‍‍‍‍‍‍‍‍‍‍‍ ‍ ‍‍‍‍‍‍‍‍‍‍‍‍‍‍‍‍‍‍‍‍‍‍‍‍‍ ‍ ‍ ‍ ‍ ‍‍‍‍‍‍‍‍‍‍‍‍‍‍‍‍‍‍‍‍‍‍‍‍ ‍ ‍ ‍ ‍ ‍‍‍‍‍‍‍‍‍‍‍‍‍‍‍‍‍‍‍‍‍‍‍‍ ‍ ‍ ‍ ‍ ‍‍‍‍‍‍‍‍‍‍‍‍‍‍‍‍‍‍‍‍‍‍‍‍ ‍ ‍ ‍ ‍ ‍‍‍‍‍‍‍‍‍‍‍‍‍‍‍‍‍‍‍‍‍‍‍‍ ‍ ‍ ‍ ‍

Which inequality is true when x = 4?

A. X+5<_3

B.x/2<3

c.9x>36

D.18<_-8

Answers


Answer is B. Is true

Which inequality is true when x = 4?

A. X+5<_3

✅B.x/2<3. 4/2 is less than 3 is true!

c.9x>36

D.18<_-8

Answer:

[tex] \sf{b. \frac{x}{2} <3}[/tex]

step by step explanation:

We will try one by one.

A. x + 5 < 3

Step 1. Subtitution Value of x

= 4 + 5 < 3

Step 2. Add by 4 with 5

= 9 < 5

A is wrong because 9 is bigger then 5

B. x/2 < 3

= 4/2 < 3

= 2 < 3

is true because 2 is smaller than 3

C. 9x > 36

9(4) > 36

36 > 36

is wrong, should 36 = 36

D. 18 < 8

is wrong because 18 is bigger than 8

3 + b/d

HELP !!!!!!!!!!!!!!!!!!!!!!!!!!!!!!!!!!!!!!!!

Answers

Fractions are mathematical expressions that relate two numbers as a quotient. Thus the answer to this question is:  [tex]\frac{3d + b}{d}[/tex]

Fractions are expressions in a form of a quotient that relates one number to another number. The two numbers can be classified as either the numerator or the denominator. The number above the division line is the numerator, while that below the line is the denominator. Some types of fractions are proper fractions, improper fractions, and mixed fractions.

A given fraction can undergo basic arithmetic operations when two or more are involved. Also, a fraction can be added to one another, or subtracted from a whole number.

The given expression in the question involves the addition of a whole number (3), to a fraction (b/d).

Given: 3 + b/d

The LCM is d, thus;

3 + b/d = [tex]\frac{3d + b}{d}[/tex]

Therefore, the required answer is: [tex]\frac{3d + b}{d}[/tex]

For more clarifications on arithmetic operations on fractions, visit: https://brainly.com/question/21561198

#SPJ1

Factor problems 1-4 by finding the GCF.

1. 4x3 – 12x2 + 4x

A. 4x(x2 – 3x + 1)
B. 4x(4x2 – 3x + 1)
C. 4x(4x2 – 3x - 1)
D. 4x(x2 – 6x + 1)
E. 2x(2x2 – 6x + 2)
F. This polynomial cannot be factored using the GCF method.
2. 9x4 + 4x3 - 27x2 + 12x

A. x(9x3 + 4x2 + 27x + 12)
B. x(9x3 + 4x2 - 27x + 12)
C. x(9x4 + 4x3 – 27x2 + 12x)
D. x4(9x3 + 4x2 + 27x + 12)
E. x(9x4 + 4x3 - 27x2 + 12x)
F. This polynomial cannot be factored using the GCF method.
3. x2 – 3x + 4

A. x(x – 3)
B. x(x + 3)
C. x(x – 1)
D. x(x – 7)
E. x(x + 7)
F. This polynomial cannot be factored using the GCF method.
4. 17x5 + 51x2 – 34

A. 17x5 + 3x2 + 2
B. 17(x3 + 3x2 + 2)
C. 17(x4 + 3x2 + 2)
D. 17(x5 + 3x2 – 2)
E. x5 + 3x2 – 2
F. This polynomial cannot be factored using the GCF method.
Factor problems 5 - 8 by grouping.

5. x2 – 2x + 1

A. (x – 1)(x – 1)
B. (x – 1)(x + 1)
C. (x + 1)(x – 1)
D. (x + 1)(x + 1)
E. (x + 2)(x – 1)
F. This polynomial cannot be factored by grouping.
6. x2 + 2x – 15

A. (x - 5)(x – 3)
B. (x + 5)(x + 3)
C. (x + 5)(x – 3)
D. (x - 5)(x + 3)
E. (x + 15)(x – 1)
F. This polynomial cannot be factored by grouping.
7. x2 – 3x – 18

A. (x + 3)(x + 6)
B. (x + 3)(x - 6)
C. (x - 3)(x - 6)
D. (x - 3)(x + 6)
E. (-x + 3)(x + 6)
F. This polynomial cannot be factored by grouping.
8. 8x2 + 47x + 28

A. This polynomial cannot be factored by grouping.
B. (3x + 7)(5x - 4)
C. (3x - 7)(5x - 4)
D. (3x + 7)(-5x + 4)
E. (-3x + 7)(-5x + 4)
F. (3x + 7)(5x + 4)

Answers

The factor of the polynomial expression 4x^3 – 12x^2 + 4x by using GCF method is  4x(x^2 - 3x + 1).

What is a factor of a number?

The factors of a given number are those numbers that are divisible by the given number without a remainder.

By using the GCF method in an algebraic, i.e. the factors of the algebraic equation will be dependent on the greatest common factor.

From the given information, we are to factorize the following expression by using the GCF method as follows.

1.

4x^3 – 12x^2 + 4x

Here; the common expression in all the three variables is 4x. So, we have:

= 4x(x^2 - 3x + 1)

Option A is correct

2.

9x^4 + 4x^3 - 27x^2 + 12x

The common expression here is (x); So,

= x(9x^3 +4x^2 - 27x + 12)

Option B is correct

3.

x^2 - 3x + 4

This polynomial expression cannot be factored.

Option F is correct.

4.

17x^5 + 51x^2 - 34

= 17(x^5 + 3x^2 - 2)

Option D is correct.

The factorization of a quadratic equation (polynomial expression) in the form (ax^2 + bx + c) by grouping results into two factors that if multiplied together result back into the original equation.

Given that:

5.

x^2 – 2x + 1

Factors are two numbers that if the multiplied result in (ac) and if added it gives us (b)

By grouping, the polynomial expression is:

= (x - 1)(x - 1)

Option A is correct

6.

x^2 + 2x - 15

= (x + 5) (x - 3)

Option C is correct

7.

x^2 – 3x – 18

=(x + 3)(x - 6)

Option B is correct

8.

8x^2 + 47x + 28

= This polynomial expression cannot be factored in as it contains rational numbers.

Option A is correct.

Learn more about factors of polynomial expressions here:
https://brainly.com/question/14314880

#SPJ1

Consider the following sample data set of 20 numbers.
24 30 4 42 40 26 23 36 12 45 29 21 34 16 47 28 32 54 19 9
1.
Create a relative frequency table and a frequency histogram with the bin width 8.
2.
Find the sample mean. (Round to the nearest hundredth.)
3.
Find the sample standard deviation

. (Round to the nearest hundredth.)

Answers

The mean = 28.55, while the standard deviation is 13.16

1. The histogram is an attachment

How to calculate for the mean of the data set

We have the following numbers

24 30 4 42 40 26 23 36 12 45 29 21 34 16 47 28 32 54 19 9

The sum of these numbers is given as

571

The total numbers = 20

The mean = 571/20

= 28.55

2 =  How to find the standard deviation

=  (24 - 28.55)2 + ... + (9 - 28.55)/20 - 1

=  3292.95/19

=  173.31315789474

s =  √173.31315789474

=  13.164845532506

Read more on mean and standard deviation here:

https://brainly.com/question/26941429

#SPJ1

Does anyone know the answer to this?

Answers

Answer: equation: y=-4x+1

slope: -4

y intercept:1

Find the missing side of the triangle. pythagorean 4 A. 337‾‾‾‾√ km B. 5‾√ km C. 193‾‾‾‾√ km D. 112‾√ km

Answers

The missing side in the given triangle is x = √193 km

Pythagoras theorem

The Pythagoras theorem is expressed as;

x^2 = a^2 + b^2

where

x is the longest side

Substitute the given side

x^2 = 12^2 + 7^2

x^2 = 144 + 49

x = √193 km

Hence the missing side in the given triangle is x = √193 km

Learn more on Pythagoras theorem here; https://brainly.com/question/654982

#SPJ1

On 1st January 2014, Rs1,000,000 was invested and on 1st January of each
successive year Rs500,000 was added to it. What sum will be accumulated by
31st December 2018 if interest is compounded each year at 10% per annum?

Answers

The sum accumulated by 31st December 2018 is Rs 4,163,060.00

What is future value?

Future value is the accumulated amount that investments would yield in future period when the amounts invested are added with the total interest earned.

Note in this case that Rs500,000  would be invested subsequently for 4 years( 2015-2018 , January 1st of each year)

We can determine the future value of each investment using the future value formula shown below:

FV=PV*(1+r)^N

PV=amount invested

r=interest=10%

N=number of years that an amount is invested(for instance, the amount invested on 1st January 2014 will earn interest in 2014,2015,2016, 2017 and 2018 i.e. for 4 years)

Total FV=1,000,000*(1+10%)^5+500,000*(1+10%)^4+500,000*(1+10%)^3+500,000*(1+10%)^2+500,000*(1+10%)^1

Total FV=Rs 4,163,060.00

Find additional guidance on future value in the link below:

https://brainly.com/question/24703884

#SPJ1

Attached as an image below.

Answers

The formula for the squirrel population is P(t) = 400 e^0.5t and the doubling time of the population is 17. 68 years

Growth constant


The growth constant k can be defined as the frequency or also known as the number of times per unit time of growing by a factor

it is also called the logarithmic return, compounded return, or the force of interest.

Note that the general formula for growth rate is given as;

[tex]P(t) = P0 e^k^t[/tex]

Where,

Po =  Initial value = 400 squirrels

r = Rate of growth

k = constant of proportionality = 0. 5 per year

t = time = 1 year

The function P(t) is given as;

P(t) = 400 e^0.5t

Note that the formula for doubling time is given as;

[tex]TD = t \frac{In 2}{In 1 + r/ 100}[/tex]

r = 4

t = 1

substitute into the formula

[tex]TD = 1 \frac{0. 693}{0. 0392}[/tex]

Doubling time  = 1 × 17. 68

Doubling time = 17. 68 years

Thus, the formula for the squirrel population is P(t) = 400 e^0.5t and the doubling time of the population is 17. 68 years

Learn more about doubling time here:

https://brainly.com/question/16407547

#SPJ1

Which inequality is represented by the given graph?
C) y ≤ -3x+6
D) y ≥ 3x+6

Answers

Answer: c, y ≤ -3x+6

Step-by-step explanation:

for example we could use a point on the graph such as (-2, 0)

now we would insert the numbers into the inequality:

y ≤ -3x+6

= 0 ≤ -3(-2) +6

= 0 ≤ 6+6

= 0 ≤ 12

which is true! therefore it is y ≤ -3x+6

16
3. A Ferris wheel with a radius of 25 m makes 2 rotations every minute.
a. State the period of the Ferris wheel.
b. Complete a tables of values showing the height above the ground for 2 rotations of the
Ferris wheel by assigning the correct variables
c. Graph 2 rotations of the Ferris wheel. (Hint: Time Vs Height above the platform)
d. State the amplitude of the Ferris wheel.

Answers

A) The Period is 30 seconds

D) The amplitude is 25 m

How to find the period and amplitude?

A) Since the Ferris wheel makes 2 rotations every minute, then we can say that the period is;

T = 1/2 = 0.5 minutes = 30 seconds

B) Let t be time in seconds and let h be height and as such, we have the formula; h = 25 - 25 cos ((2π/30)t)

Thus, the table is attached showing input values of t and their corresponding h values.

C) The graph of the table above is as attached below.

D) The amplitude is;

Amplitude = (Maximum Point - Minimum Point)/2

Amplitude = (50 - 0)/2

Amplitude = 25

Read more about Amplitude & Period at; https://brainly.com/question/23713359

#SPJ1

Please draw the number line and show me if possible:)

Answers

The solution to the given inequality expression is expressed as x ≥ 6

Solving inequalities

Given the inequality expression shown below;

5x-9/7≥3

Cross multiply

(5x -9)≥7(3)

Expand

5x-9≥21

Add 9 to both sides

5x ≥ 21 + 9

5x ≥ 30

x ≥6

The solution to the given inequality expression is expressed as x ≥ 6

Learn more on inequalities here: https://brainly.com/question/24372553

#SPJ1


A stadium has 53,000 seats. Seats sell for $25 in Section A, $20 in Section B, and $15 in Section C. The number of seats in Section A
equals the total number of seats in Sections B and C. Suppose the stadium takes in $1,131,000 from each sold-out event. How many seats
does each section hold?

Answers

The number section A, section B and section C seats sold are 26500, 14200 and 12300 respectively.

How to use equation to find the total number of seat in each section?

The stadium has 53,000 seats.

Seats sell for $25 in Section A, $20 in Section B, and $15 in Section C.

The number of seats in Section A equals the total number of seats in Sections B and C.

Therefore,

a =  b + c

a + b + c = 53000

b + c + b + c = 53000

2b + 2c = 53,000

25a + 20b + 15c = 1131000

25(b + c) + 20b + 15c  = 1131000

25b + 25c + 20b + 15c = 1131000

45b + 40c = 1131000

Hence,

2b + 2c = 53000

45b + 40c = 1131000

40b + 40c  = 1060000

45b + 40c = 1131000

-5b = - 71000

b = - 71000 / -5

b = 14,200

Therefore,

2(14,200) + 2c = 53000

2c = 53000 - 28400

2c = 24600

c = 24600 / 2

c = 12300

Hence,

a =  b + c

a = 14200 + 12300

a = 26500

learn more on equation here: https://brainly.com/question/17447452

#SPJ1

Find sec, tan 0, and sin 0, where is the angle shown in the figure.
Give exact values, not decimal approximations.
0
4
5

Answers

The exact values of sec θ, tan θ and sin θ are 6.4/4, 5/4 and 5/ 6.4 respectively.

How to determine the trigonometric ratios

From the figure, we have the values of the sides to be;

opposite side = 5Adjacent side = 4

Let's use Pythagorean theorem to find the hypotenuse(x)

Hypotebuse square = opposite side square + adjacent side square

Substitute the values deduced from the figures given

x² = 5² + 4²

x² = 25 + 16

x² = 41

x = [tex]\sqrt{41}[/tex]

x = 6. 4

sin θ = opposite/hypotenuse

Substitute the values

sin θ = 5/ 6. 4

tan θ = opposite/ adjacent

Substitute the values

tan θ = 5/ 4

sec θ = hypotenuse/ adjacent

substitute the values

sec θ = 6. 4/ 4

Thus, the exact values of sec θ, tan θ and sin θ are 6.4/4, 5/4 and 5/ 6.4 respectively.

Learn more about trigonometric identities here:

https://brainly.com/question/7331447

#SPJ1

Calculate the perimeter and area of this shape.
Perimeter =
Area =
7.5 cm
6 cm
2 cm
3 cm
2 cm

Answers

area = 25.5 cm²

Step-by-step explanation:

i have shown in the image you have to rotate it to see how i worked the area out

i am very sorry i cant work the perimeter out since i have no idea what the length of the two sides of the triangle are . if you need help on understanding how i worked out the area please let me know .

hopefully i was able to help you in half of the question i am sorry for not helping on the perimeter :)

Answer:

Step-by-step explanation:

Perimeter=25

50 POINTS!!!!!!!!!!!!!!!!!!!!!!!! SOLVE THIS QUICKLY WITH AN EXPLANATION

Answers

Answer:

x = 22°

Step-by-step explanation:

Between two parallel lines, the opposite inside angles will be congruent. So, x is equal to angle V. The sum of the angles of any triangle will be 180°. Therefore, you can find the value of x by setting all the angles of the triangle SRV equal to 180°.

S = 5x + 4

R = 44

V = x

S + R + V = 180                                         <----- General equation

(5x + 4) + 44 + x = 180                              <----- Insert values

5x + 48 + x = 180                                      <----- Add 4 and 44

6x + 48 = 180                                           <----- Add 5x and x

6x = 132                                                    <----- Subtract 48 from both sides

x = 22                                                      <----- Divide both sides by 6

Solve p/-1+ 17 = 13 for p.

Answers

Answer: 208

Step-by-step explanation:

[tex]\frac{p}{-1+17} =13[/tex]

[tex]\frac{p}{16} =13[/tex]

[tex](16)\frac{p}{16} =13(16)[/tex]

[tex]p=13*16[/tex]

[tex]p=208[/tex]

[tex]\large\displaystyle\text{$\begin{gathered}\sf \frac{p}{-1}+17=13 \end{gathered}$}[/tex]

[tex]\large\displaystyle\text{$\begin{gathered}\sf \bf{Apply \ the \ properties \ of \ fractions: \frac{a}{-b}=-\frac{a}{b}. } \end{gathered}$}[/tex]

[tex]\large\displaystyle\text{$\begin{gathered}\sf \bf{=-\frac{P}{1} } \end{gathered}$}[/tex]

[tex]\large\displaystyle\text{$\begin{gathered}\sf \bf{Apply\:the\:rule \:\frac{a}{1}=a} \end{gathered}$}[/tex]

[tex]\large\displaystyle\text{$\begin{gathered}\sf \bf{=-P} \end{gathered}$}[/tex][tex]\large\displaystyle\text{$\begin{gathered}\sf \bf{-P+17=13} \end{gathered}$}[/tex]

[tex]\large\displaystyle\text{$\begin{gathered}\sf \bf{Subtract \ 17 \ from \ both \ sides. } \end{gathered}$}[/tex]

[tex]\large\displaystyle\text{$\begin{gathered}\sf \bf{-P+17-17=13-17} \end{gathered}$}[/tex]

[tex]\large\displaystyle\text{$\begin{gathered}\sf \bf{Simplify } \end{gathered}$}[/tex]

[tex]\large\displaystyle\text{$\begin{gathered}\sf \bf{-P=-4} \end{gathered}$}[/tex]

[tex]\large\displaystyle\text{$\begin{gathered}\sf \bf{Divide\:both\:sides\:by\:-1} \end{gathered}$}[/tex]

[tex]\large\displaystyle\text{$\begin{gathered}\sf \bf{\frac{-p}{1}=\frac{-4}{-1} } \end{gathered}$}[/tex]

[tex]\large\displaystyle\text{$\begin{gathered}\sf \bf{Simplify } \end{gathered}$}[/tex]

[tex]\boxed{\large\displaystyle\text{$\begin{gathered}\sf \bf{P=4} \end{gathered}$}}[/tex]
Other Questions
at the end of the accounting period, meyer company has a balance of $27,000 in the service expense account. what entry should be made to close the adervertis expense account? Why do you need to use a statistical test (e.g., the chi-square test) to compare the observed genotype frequencies in a population to those expected under Hardy-Weinberg equilibrium? to distinguish between natural selection and other factors that could cause a population to deviate from Hardy-Weinberg equilibrium because it is impossible to know what the observed genotype frequencies are in a population to assess whether the deviation of observed from expected frequencies is likely due to chance because the frequencies expected under Hardy-Weinberg equilibrium are not accurate An atom with an atomic number of 14 will have electrons in its valence shell. O 8 O 10 O 2 O 14 DANO suppose that for every positive integer i, all the entries in the ith row and ith column of the adjacency matrix of a graph are 0. what can you conclude about the graph? For the overbooking strategy, why is the optimal number of units (i.e. airline seats, hotel rooms) overbooked is not equal to the expected number of no-shows?a. Variability in the number of no-shows makes the number difficult to calculate.b. The costs associated with overbooking and under-booking are not equal.c. It is illegal to excessively overbook.d. Hotels and airlines reserve some spots for important or emergency situations. For the output levels in Table 21.4, the minimum of the average total cost curve occurs at a production rate of Select one: A. 3 units per day. B. Zero units per day. C. 4 units per day. D. 2 units per day. which term is used to describe the notion that the nerves that branch out from the spinal cord and brain become less efficient Which one of the following is true about the Cisco core layer in the three-tier design?A. Never do anything to slow down traffic. This includes making sure you don't use access lists, perform routing between virtual local area networks, or implement packet filtering.B. It's best to support workgroup access here.C. Expanding the core, eg adding routers as the internet work grows. Is highly recommended as a first step.D. All cables from the core must connect to the TOR Assume the following process is used in production of a product. There are 3 workers in this process; each task is done by 1 worker. Assuming the process works at full capacity, utilization of the worker performing TASK A is: TASKA TASK B TASK C Flag question 3 min./Unit 6 min./Unit 1 min./Unit Select one: O a. 16.66% O b. None of the provided answers O c. 55.55% O d. 33.33% O e. 100% O f. 50% when evaluating an investment in a mortgage pass-through security, which of the following is not one of the characteristics of the underlying mortgage pool that should be considered? a. set a data validation rule for the range b5:f5 that allows only whole number values greater than 0. why have bank runs in the united states been relatively scarce since the 1930s? Calculate the multiplier Question Find the money multiplier associated with a reserve requirement of 0.12 if banks hold no excess reserves and consumers hold no cash. Round your answer to two decimal places. Provide your answer below: MM- shift in the demand curve of labor due to increase competition from chines eimport are comparable to shift in the demand curve due to FILL IN THE BLANK according to komisar, what distinguishes the silicon valley is not its successes, but _____. an employers accounting for a defined benefit pension plan is based on the assumption that such a plan is part of an employees compensation, and that the compensation expense is incurred when the T or F. Sourcing occurs when the current flows out of the PLC port and sinking occurs when the current flows into the PLC port to examine whether children's popularity was stable or changed across the years, dr. clique followed a group of children from ages 5 to 18 years. this is an example of a ________ design. You have taken a long position in a call option on IBM common stock. The option has an exercise price of $150 and IBM's stock currently trades at $153. The option premium is $5 per contract.a. How much of the option premium is due to intrinsic value versus time value?b. What is your net profit on the option if IBMs stock price increases to $163 at expiration of the option and you exercise the option?c. What is your net profit if IBMs stock price decreases to $143? The firm's short-run supply curve is a(n)____ -sloping curve that begins at ____ average variable cost.Click the answer you think is right. upward; maximum downward; maximum downward: minimum upward; minimum